8 votos

Sobre el ejercicio 3.26 de Atiyah-Macdonald

Estoy intentando probar el ejercicio 3.26 sobre Atiyah-Macdonlad:

Sea $(B_{\alpha},g_{\alpha \beta})$ un sistema directo de anillos y $B$ el límite directo. Para cada $\alpha$ , dejemos que $f_{\alpha}:A\rightarrow B_{\alpha}$ sea un homomorfismo de anillo tal que $g_{\alpha \beta}\circ f_{\alpha}=f_{\beta}$ siempre que $\alpha\leq \beta$ . Entonces $f_{\alpha}$ inducir $f:A\rightarrow B$ . Demuestre que $$f^{\ast}(\mathrm{Spec}(B))=\bigcap f_{\alpha}^{\ast}(\mathrm{Spec}(B_{\alpha}))$$


Siguiendo la pista, descubrí $$\begin{aligned}\mathfrak{p}\notin f^{\ast}(\mathrm{Spec}(B)) &\Leftrightarrow \varinjlim(B_{\alpha}\otimes_A k(\mathfrak{p}))=0\end{aligned}$$ y $$\begin{aligned}\mathfrak{p}\notin \bigcap f_{\alpha}^{\ast}(\mathrm{Spec}(B_{\alpha}))&\Leftrightarrow B_{\alpha}\otimes_A k(\mathfrak{p})=0 \text{ for some }\alpha\end{aligned}$$

Por el ejercicio 2.21 sobre Atiyah-Macdonald, tenemos

$$ \varinjlim(B_{\alpha}\otimes_A k(\mathfrak{p}))=0 \Rightarrow B_{\alpha}\otimes_A k(\mathfrak{p})=0 \text{ for some }\alpha$$

Pero no tengo ni idea de cómo demostrar lo contrario, que es cierto según Atiyah-Macdonald.

Como el límite directo es la suma directa módulo de algo, consideremos la suma directa de $B_{\alpha}$ . Se eliminarán los anillos cero. La parte cociente es desconocida pero intuitivamente no debería ser la suma directa del resto de anillos distintos de cero. ¿El hecho de que $B_{\alpha}\otimes_A k(\mathfrak{p})$ es un $k$ -¿importa el módulo aquí?

Cualquier sugerencia o respuesta será bienvenida.

2 votos

El submódulo que cociente a cabo es generado por esas relaciones a través de sistema directo. En particular, si desaparece en una posición, desaparecerá en el límite directo bajo cociente. Aquí estás usando implícitamente una afirmación más fuerte por límite directo conmutando a través de suma directa. Puesto que $B_a$ son todos $A-$ módulos, puede asumir la estructura del módulo y utilizar la estructura del módulo para deducir.

0 votos

@user45765 ¿Podrías elaborar un poco tu respuesta o incluso escribir una respuesta al respecto? ¡¡¡Gracias!!!

0 votos

He cometido un error tipográfico. Debería haber dicho límite directo conmutando a través del producto tensorial. Basta con tratar el límite como $A-$ en su lugar. (Esto es más fácil para que la imagen sea más clara y la estructura de multiplicación no es realmente importante aquí). Entonces sabes que cualquier límite directo de $M_i$ como $A-$ módulos es $\frac{\oplus_iM_i}{\sim}$ donde $\sim$ identifica la imagen bajo el mapa de transición $M_i\to M_j$ (El cociente es en realidad un submódulo generado por esos elementos.) Entonces se muestra el límite directo conmutando con el producto tensorial. Si desaparece en una posición, tiene que ser $0$ por mapa de módulos.

2voto

Aolong Li Puntos 11

En general, es no cierto que $\varinjlim B_{\alpha}=0$ sólo si $B_{\alpha}=0$ para algunos $\alpha$ .

Consideremos el contraejemplo trivial: el sistema directo consiste en $k$ y $0$ y $$0 \rightarrow 0 \rightarrow 0\rightarrow \cdots$$ $$ k \xrightarrow{\mathrm{id}} k\xrightarrow{\mathrm{id}}k\xrightarrow{\mathrm{id}}\cdots $$ donde $k$ es un campo (también un $k$ -álgebra). Sin embargo, tenemos $B_{\alpha}=0$ para algunos $\alpha$ y $\varinjlim B_{\alpha}=k~\sqcup \{0\}$ que obviamente no es un anillo cero.

De hecho, podemos demostrar lo contrario fácilmente. Siguiendo la pista, ya demostramos que $$f^{\ast}(\mathrm{Spec}(B))\supset\bigcap f_{\alpha}^{\ast}(\mathrm{Spec}(B_{\alpha})).$$ Ahora, tenemos que mostrar $$f^{\ast}(\mathrm{Spec}(B))\subset\bigcap f_{\alpha}^{\ast}(\mathrm{Spec}(B_{\alpha})).$$ Toma un $\mathfrak{p}\in f^{\ast}(\mathrm{Spec}(B))$ es decir, existe $\mathfrak{q}$ tal que $f^{-1}(\mathfrak{q})=\mathfrak{p}$ . Supongamos que los morfismos de proyecto $$\mu_{\alpha}:B_{\alpha}\rightarrow B$$ y tenemos $f=\mu_{\alpha}\circ f_{\alpha}$ para cualquier $\alpha$ . Tomemos el ideal primo $\mathfrak{p}_{\alpha}=\mu_{\alpha}^{-1}(\mathfrak{q})$ entonces $$f^{\ast}_{\alpha}(\mathfrak{q}_{\alpha})=f_{\alpha}^{-1}\mu_{\alpha}^{-1}(\mathfrak{q})=(\mu_{\alpha}\circ f_{\alpha})^{-1}(\mathfrak{q})=f^{-1}(\mathfrak{q})=\mathfrak{p}$$ De ello se deduce que $$\mathfrak{p}\in \bigcap f_{\alpha}^{\ast}(\mathrm{Spec}(B_{\alpha})). $$


Es decir, la equivalencia que pretendía el autor, $$\varinjlim(B_{\alpha}\otimes_A k(\mathfrak{p}))=0 \Leftrightarrow B_{\alpha}\otimes_A k(\mathfrak{p})=0 \text{ for some }\alpha$$

es verdadera, por la equivalencia anterior. Pero no sé cómo demostrar la dirección inversa directamente hasta ahora.

1 votos

No entiendo el contraejemplo que propones. No hay ningún morfismo de anillos $0 \to k$ si no me equivoco: cualquier homomorfismo de anillo de este tipo tendría que tomar $0$ a la identidad aditiva y multiplicativa en $k$ simultáneamente, una contradicción.

0 votos

@AlexWertheim ¡buena observación! Déjame pensarlo.

0 votos

@AlexWertheim He editado mi respuesta. Espero que esta vez no he hecho ningún error estúpido de nuevo.

1voto

Z Kane Puntos 28

En general, $\varinjlim B_{\alpha}=0$ sólo si $B_{\alpha}=0$ para algunos $\alpha$ .

A continuación se demuestra que $\boxed{ \text{if $ B_\alpha=0 $ for some $ \alfa $, then $ \varinjlim B_{{alfa}=0 $}}:$

Recordemos por el Ejercicio 2.21, la identidad multiplicativa $1\in \varinjlim B_{\alpha}$ es la imagen de $1_\alpha\in B_\alpha$ bajo el homomorfismo canónico de anillo $g_\alpha: B_\alpha \to \varinjlim B_\alpha$ . Desde $B_\alpha=0$ tenemos que $1_\alpha=0_\alpha$ . Así que.., $1=g_\alpha(1_\alpha)=g_\alpha(0_\alpha)=0$ .


A continuación se muestra un innecesario cálculo que muestra que todo en $\varinjlim B_\alpha$ es igual a $0$ .

Sea $u\in \varinjlim B_\alpha$ . Entonces, por el Ejercicio 2.15, existe $v\in B_\beta$ para algunos $\beta$ tal que $u=g_\beta(v)\in \mathrm{Im}(g_\beta)$ .

Como el conjunto índice es dirigido, puede encontrar un índice $\gamma\geq \alpha,\beta$ . Así, \begin{align*} u &= 1 \cdot u \\&= g_\alpha(1_\alpha) \cdot g_\beta(v) \\&= g_\alpha(0_\alpha) \cdot g_\beta(v) \\&= g_\gamma(g_{\alpha\gamma}(0_\alpha) \cdot g_{\beta\gamma}(v)) \\&= g_\gamma(0_\gamma \cdot g_{\beta\gamma}(v)) \\&= g_\gamma(0_\gamma) \\&= 0 \end{align*} lo que implica que $\varinjlim B_\alpha = 0$ .

i-Ciencias.com

I-Ciencias es una comunidad de estudiantes y amantes de la ciencia en la que puedes resolver tus problemas y dudas.
Puedes consultar las preguntas de otros usuarios, hacer tus propias preguntas o resolver las de los demás.

Powered by:

X